Você está na página 1de 28

Junior problems

J367. Let a and b be positive real numbers. Prove that


1
3
1
4
4
+
+

+
.
4a a + b 4b
3a + b a + 3b
Proposed by Nguyen Viet Hung, Hanoi University of Science, Vietnam

Solution by Nermin Hodzic, Dobosnica, Bosnia and Herzegovina


We can rewrite the inequality as
a+b
3
16(a + b)

4ab
a+b
4ab + 3(a + b)2


 a+b
3
4ab + 3(a + b)2
+
16(a + b).
4ab
a+b
Using the Cauchy-Schwarz inequality we have
 

2

 a+b
3
2
+
4ab + 3(a + b)

a + b + 3 a + b = 16(a + b).
4ab
a+b
Equality holds if and only if a = b.

Also solved by Henry Ricardo, New York Math Circle, Tappan, NY, USA; Bobojonova Latofat, Academic
lyceum S.H.Sirojiddinov, Tashkent, Uzbekistan; ngel Plaza, Department of Mathematics, University of Las
Palmas de Gran Canaria, Spain; Joel Schlosberg, Bayside, NY, USA; Albert Stadler, Herrliberg, Switzerland; Corneliu Mnescu-Avram, Transportation High School, Ploieti, Romania; Yong Xi Wang, Affiliated
High School of Shanxi University; Adnan Ali, Student in A.E.C.S-4, Mumbai, India; Ioan Viorel Codreanu,
Satulung, Maramures, Romania; David E. Manes, Oneonta, NY, USA; Alan Yan, Princeton Junction, NJ,
USA; David Stoner, Harvard University, Cambridge, MA, USA; Petros Panigyrakis, Evaggeliki Gymnasium,
Athens, Greece; Nicuor Zlota Traian Vuia Technical College, Focani, Romania; Joachim Studnia, Lyce
Condorcet, Paris, France; Dimitris Avramidis, Evaggeliki Gymnasium, Athens, Greece; Duy Quan Tran,
University of Health Science, Ho Chi Minh City, Vietnam; Paul Revenant, Lyce Champollion, Grenoble,
France; Polyahedra, Polk State College, FL, USA; Moubinool Omarjee Lyce Henri IV, Paris, France; Brian
Bradie, Department of Mathematics, Christopher Newport University, Newport News, VA, USA; Daniel Lasaosa, Pamplona, Spain; Evgenidis Nikolaos, M.N. Raptou High School, Larissa, Greece; Arkady Alt, San
Jose, CA, USA; WSA.

Mathematical Reflections 2 (2016)

J368. Find the best constants and such that <

x
y
+
for all x, y (0, ).
2x + y x + 2y

Proposed by ngel Plaza, Universidad de Las Palmas de Gran Canaria, Spain

Solution by Polyahedra, Polk State College, FL, USA


By the AM-HM inequality,
x
y
+
= 2 (x + y)
2x + y x + 2y

1
1
+
2x + y x + 2y


2

4(x + y)
2
= ,
(2x + y) + (x + 2y)
3

with equality if and only if x = y. Hence = 23 . On the other hand, assume that x y. Then 2(2x + y) >
2x + y x + 2y, thus


x
1
y
1
1
> 0,
+
=y

2x + y 2 x + 2y
x + 2y 2(2x + y)
and this expression 0 as y 0 for fixed x. Hence = 12 .
Also solved by Paul Revenant, Lyce Champollion, Grenoble, France; WSA; Hyun Min Victoria Woo,
Northfield Mount Hermon School, Mount Hermon, MA, USA; Ji Eun Kim, Tabor Academy, Marion, MA,
USA; Yong Xi Wang, Affiliated High School of Shanxi University; Corneliu Mnescu-Avram, Transportation
High School, Ploieti, Romania; Albert Stadler, Herrliberg, Switzerland; David Stoner, Harvard University,
Cambridge, MA, USA; Bobojonova Latofat, Academic lyceum S.H.Sirojiddinov, Tashkent, Uzbekistan; Joel
Schlosberg, Bayside, NY, USA; Jorge Ledesma, Faculty of Sciences UNAM, Mexico City, Mexico; Alan
Yan, Princeton Junction, NJ, USA; Henry Ricardo, New York Math Circle, Tappan, NY, USA; AN-anduud
Problem Solving Group, Ulaanbaatar, Mongolia; Adnan Ali, Student in A.E.C.S-4, Mumbai, India; Daniel
Lasaosa, Pamplona, Spain; Nermin Hodzic, Dobosnica, Bosnia and Herzegovina; Brian Bradie, Department
of Mathematics, Christopher Newport University, Newport News, VA, USA; Arkady Alt, San Jose, CA,
USA.

Mathematical Reflections 2 (2016)

J369. Solve the equation


r

1
1
1
+
= x+ .
x+1
x
x+1

1+

Proposed by Nguyen Viet Hung, Hanoi University of Science, Vietnam

Solution by Alessandro Ventullo, Milan, Italy


Clearly, x > 0. The given equation can be written as

x+2
1
x+1

+
= ,
x
x+1
x+1
i.e.

x2 + 2x + x = (x + 1) x + 1.

The last equation can be written as

which gives

x+

x
= x + 1,
x+21

x+1=

(x + 2)(x + 1).

Squaring both sides and reordering, we get


p
x2 + x + 1 = 2 x2 + x,
i.e.
p
( x2 + x 1)2 = 0.
The last equation is equivalent to

x2

1 +
+ x 1 = 0, which solving for x > 0 gives x =
2

Also solved by Moubinool Omarjee Lyce Henri IV , Paris France; ngel Plaza, Department of Mathematics, University of Las Palmas de Gran Canaria, Spain; Bobojonova Latofat, Academic lyceum S.H.Sirojiddinov,
Tashkent, Uzbekistan; Yong Xi Wang, Affiliated High School of Shanxi University; Hyun Min Victoria Woo,
Northfield Mount Hermon School, Mount Hermon, MA, USA; Joel Schlosberg, Bayside, NY, USA; Ji Eun
Kim, Tabor Academy, Marion, MA, USA; Joe Currier, College at Brockport, SUNY, USA; Adnan Ali, Student in A.E.C.S-4, Mumbai, India; Vicente Vicario Garca, Sevilla, Spain; David E. Manes, Oneonta, NY,
USA; Alan Yan, Princeton Junction, NJ, USA; David Stoner, Harvard University, Cambridge, MA, USA;
Corneliu Mnescu-Avram, Transportation High School, Ploieti, Romania; WSA; Nicuor Zlota Traian
Vuia Technical College, Focani, Romania; Duy Quan Tran, University of Health Science, Ho Chi Minh
City, Vietnam; Henry Ricardo, New York Math Circle, Tappan, NY, USA; Arpon Basu; Paul Revenant,
Lyce Champollion, Grenoble, France; Polyahedra, Polk State College, FL, USA; Brian Bradie, Department
of Mathematics, Christopher Newport University, Newport News, VA, USA; Albert Stadler, Herrliberg, Switzerland; Daniel Lasaosa, Pamplona, Spain; Evgenidis Nikolaos, M.N. Raptou High School, Larissa, Greece;
Nermin Hodzic, Dobosnica, Bosnia and Herzegovina; Antoine Faisant, Lyce Pierre de Fermat, France;
Arkady Alt, San Jose, CA, USA.

Mathematical Reflections 2 (2016)

J370. Triangle ABC has sides lengths BC = a, CA = b, AB = c. If


(a2 + b2 + c2 )2 = 4a2 b2 + b2 c2 + 4c2 a2 ,
find all possible values of A.
Proposed by Adrian Andreescu, Dallas, TX, USA

Solution by Nikolaos Evgenidis, M.N. Raptou High School, Larissa, Greece


Expanding the LHS and doing the maths we have
a4 2a2 b2 + b4 + b2 c2 + c4 2c2 a2 = 0.
By adding b2 c2 to both sides, the above relation can be written as
(b2 + c2 a2 )2 = b2 c2 (b2 + bc + c2 a2 )(b2 bc + c2 a2 ) = 0.
Hence, either b2 + bc + c2 = a2 or b2 bc + c2 = a2 .
But from the Law of Cosines we know a2 = b2 + c2 2bc cos A. This, combined with the above equations,
implies either:
bc(1 + 2 cos A) = 0. Then cos A = 21 A = 120o since 0 A 180
or
bc(1 2 cos A) = 0. Then cos A =

1
2

A = 60o since 0 A 180.

Therefore, the possible values of A are 60o or 120o .


Also solved by Moubinool Omarjee, Lyce Henri IV, Paris, France; Bobojonova Latofat, Academic lyceum
S.H.Sirojiddinov, Tashkent, Uzbekistan; Corneliu Mnescu-Avram, Transportation High School, Ploieti, Romania; Yong Xi Wang, Affiliated High School of Shanxi University; Jorge Ledesma, Faculty of Sciences
UNAM, Mexico City, Mexico; Hyun Min Victoria Woo, Northfield Mount Hermon School, Mount Hermon,
MA, USA; Ji Eun Kim, Tabor Academy, Marion, MA, USA; Andreas Charalampopoulos, 4th Lyceum of Glyfada, Glyfada, Greece; Helen Kain, student, College at Brockport, SUNY; Adnan Ali, Student in A.E.C.S-4,
Mumbai, India; Vicente Vicario Garca, Sevilla, Spain; Alan Yan, Princeton Junction, NJ, USA; David
Stoner, Harvard University, Cambridge, MA, USA; WSA; Nicuor Zlota Traian Vuia Technical College,
Focani, Romania; Duy Quan Tran, University of Health Science, Ho Chi Minh City, Vietnam; Panigyraki Chrysoula, Evaggeliki Gymnasium, Athens, Greece; Dimitris Avramidis, Evaggeliki Gymnasium, Athens,
Greece; Paul Revenant, Lyce Champollion, Grenoble, France; Polyahedra, Polk State College, FL, USA;
Adam Krause, student, College at Brockport, SUNY; Alok Kumar; Albert Stadler, Herrliberg, Switzerland;
Daniel Lasaosa, Pamplona, Spain; Nermin Hodzic, Dobosnica, Bosnia and Herzegovina; Arkady Alt, San
Jose, CA, USA.

Mathematical Reflections 2 (2016)

J371. Prove that for all positive integers n,









n+3
n+4
n+5
+6
+ 90
2
4
6

is the sum of two perfect cubes.


Proposed by Alessandro Ventullo, Milan, Italy

Solution by Adnan Ali, Student in A.E.C.S-4, Mumbai, India


It is easily verified that






n+3
n+4
n+5
+6
+ 90
= 13 + a3n ,
2
4
6

(1)

(n + 4)(n + 1)
where an =
. Moreover an is an integer as n + 4 and n + 1 are always of different parities. (1)
2
may be verified as follows






n+3
n+4
n+5
+6
+ 90
2
4
6
4(n + 3)(n + 2) 2(n + 4)(n + 3)(n + 2)(n + 1) (n + 5)(n + 4)(n + 3)(n + 2)(n + 1)n
+
+
8
8
8
!
(n + 3)(n + 2)
1
1
=
1 + (n + 4)(n + 1) + (n + 5)(n + 4)(n + 1)n = (an + 1)(1 + an + an (an 2)),
2
2
4
=

and the proof is complete.


Also solved by Moubinool Omarjee Lyce Henri IV, Paris, France; ngel Plaza, Department of Mathematics, University of Las Palmas de Gran Canaria, Spain; Bobojonova Latofat, Academic lyceum S.H.Sirojiddinov,
Tashkent, Uzbekistan; Corneliu Mnescu-Avram, Transportation High School, Ploieti, Romania; Yong Xi
Wang, Affiliated High School of Shanxi University; Jorge Ledesma, Faculty of Sciences UNAM, Mexico
City, Mexico; Hyun Min Victoria Woo, Northfield Mount Hermon School, Mount Hermon, MA, USA; Ji
Eun Kim, Tabor Academy, Marion, MA, USA; Adam Krause, student, College at Brockport, SUNY; Joel
Schlosberg, Bayside, NY, USA; Joe Currier, student, College at Brockport, SUNY; Vicente Vicario Garca, Sevilla, Spain; David E. Manes, Oneonta, NY, USA; Alan Yan, Princeton Junction, NJ, USA; David
Stoner, Harvard University, Cambridge, MA, USA; Arpon Basu, AECS-4, Mumbai, India; Paul Revenant,
Lyce Champollion, Grenoble, France; Polyahedra, Polk State College, FL, USA; AN-anduud Problem Solving Group, Ulaanbaatar, Mongolia; Albert Stadler, Herrliberg, Switzerland; Brian Bradie, Department of
Mathematics, Christopher Newport University, Newport News, VA, USA; Daniel Lasaosa, Pamplona, Spain;
Nermin Hodzic, Dobosnica, Bosnia and Herzegovina.

Mathematical Reflections 2 (2016)

J372. In triangle ABC,

<ABC<
sin

5
7 .

Prove that

7A
7B
7C
7A
7B
7C
sin
+ sin
> cos
cos
+ cos
.
4
4
4
4
4
4
Proposed by Adrian Andreescu, Dallas, TX, USA

Solution by Alan Yan, Princeton Junction, NJ, USA


Let X =

7A
4 ,Y

7B
4 ,Z

7C
4 .

We have that

<XY Z<

5
4 .

Rearrange the original equality as

(sin X cos X) (sin Y cos Y ) + (sin Z cos Z) > 0

2
2
2
(sin X cos X)
(sin Y cos Y ) +
(sin Z cos Z) > 0
2
2
2






sin Y
+ sin Z
>0
sin X
4
4
4

Let U = X 4 , V = Y 4 , W = Z 4 . We have that U + V + W = and 0 < U V W .


So, there exists a triangle with angles U, V, W . Let the circumradius of this triangle be R and let the sides
corresponding to U, V, W be u, v, w, respectively. It suffices to prove that sin U sin V + sin W > 0.
Indeed,
sin U sin V + sin W > 0 2R(sin U sin V + sin W ) > 0 u v + w > 0,
which is just the triangle inequality. 
Also solved by Hyun Min Victoria Woo, Northfield Mount Hermon School, Mount Hermon, MA, USA; Ji
Eun Kim, Tabor Academy, Marion, MA, USA; Yong Xi Wang, Affiliated High School of Shanxi University;
Daniel Lasaosa, Pamplona, Spain; Nermin Hodzic, Dobosnica, Bosnia and Herzegovina; Polyahedra, Polk
State College, FL, USA; David Stoner, Harvard University, Cambridge, MA, USA.

Mathematical Reflections 2 (2016)

Senior problems

S367. Solve in positive real numbers the system of equations

(x3 + y 3 )(y 3 + z 3 )(z 3 + x3 ) = 8,


x2
y2
z2
3

+
+
= .
x+y y+z z+x
2
Proposed by Nguyen Viet Hung, Hanoi University of Science, Vietnam

Solution by David Stoner, Harvard University, Cambridge, MA, USA


X x2
X y2
X
(x y) = 0. It follows that the second
First, note that
=
, as their difference is
x+y
x+y
cyc
cyc
cyc
X x2 + y 2
condition is equivalent to
= 3.
x+y
cyc
3

+b 3
Now we claim that for positive reals a, b, the inequality a +b
( aa+b
) holds, with equality occurring
2
2
2
3
if and only if a = b. Indeed, this follows from the identity 2(a +b ) (a+b)3 (a3 +b3 ) = (ab)4 (a2 +b2 +ab).

Using this, we see that, for any x, y, z satisfying the system, we have by this claim and AM-GM:
3=

X x2 + y 2

x+y
s
 2
 2

x2 + y 2
y + z2
z + x2
3
3
x+y
y+z
z+x
s
 3
 3

x3 + y 3
y + z3
z + x3
9
3
2
2
2
= 3.

Therefore, equality must hold in the second inequality, which by our claim implies x = y = z. Plugging this
X x2 + y 2
into
= 3 gives 3x = 3, so x = y = z = 1 is the sole solution to this system; it works, so we are
x
+
y
cyc
done.
Also solved by Li Zhou, Polk State College, Winter Haven, FL, USA; Sutanay Bhattacharya, Bishnupur
High School, West Bengal, India; Nermin Hodzic, Dobosnica, Bosnia and Herzegovina.

Mathematical Reflections 2 (2016)

S368. Determine all positive integers n such that (n) = n + 55, where (n) denotes the sum of the divisors
of n.
Proposed by Alessandro Ventullo, Milan, Italy

Solution by Li Zhou, Polk State College, Winter Haven, FL, USA


Write the prime factorization of n as = pe11 pekk .


If k = 1, then 2 33 = 54 = (n) n 1 = p1 1 + + pe11 2 , which means p1 = 2, an impossibility.
If k 4, then

(n) = (1 + p1 + + pe11 ) 1 + pk + + pekk > n + p1 p2 p3 + p2 p3 p4 + p3 p4 p1 + p4 p1 p2
n + 2 3 5 + 3 5 7 + 5 7 2 + 7 2 3 > n + 55.
Next, consider k = 3. If n is odd, then
(n) > n + p1 p2 + p2 p3 + p3 p1 n + 3 5 + 5 7 + 7 3 > n + 55.
If n is even, then p1 = 2. Since 55 = (n) n, (n) must be odd. Hence e2 , e3 must be even, thus


(n) > n + 1 + 3 + 32 1 + 5 + 52 > n + 55.
Finally, consider k = 2. If e1 = e2 = 1, then 54 = (n) n 1 = p1 + p2 and we get {p1 , p2 } = {7, 47},
{11, 43}, {13, 41}, {17, 37}, {23, 31}. If (e1 , e2 ) = (2, 1), then

(n) n = 1 + p1 + p21 (1 + p2 ) p21 p2 = (1 + p1 )(1 + p2 ) + p21 ,
which is greater than 55 for p1 > 5 and cannot equal 55 for p1 = 2, 3, 5. If e1 = e2 = 2, then

(n) n = 1 + p2 + p22 (1 + p1 ) + p21 (1 + p2 ) 55,
with equality if and only if {p1 , p2 } = {2, 3}. If e1 3, then

(n) n 1 + p1 + p21 (1 + p2 ) + p31 ,
which is greater than 55 for p1 3. But if p1 = 2, then e2 must be even for (n) to be odd. Thus


(n) n 1 + p1 + p21 1 + p2 + p22 7 13 > 55.
In conclusion, the solutions are n = 7 47 = 329, 11 43 = 473, 13 41 = 533, 17 37 = 629, 23 31 = 713,
and 22 32 = 36.
Also solved by Khurshid Juraev, Academic lyceum named after S.H.Sirojiddinov, Tashkent, Uzbekistan;
David Stoner, Harvard University, Cambridge, MA, USA; Latofat Bobojonova, Academic lyceum S.H.Sirojiddinov,
Tashkent, Uzbekistan; David E. Manes, Oneonta, NY, USA; Adnan Ali, Student in A.E.C.S-4, Mumbai, India; Nermin Hodzic, Dobosnica, Bosnia and Herzegovina; Albert Stadler, Herrliberg, Switzerland; Moubinool
Omarjee, Lyce Henri IV, Paris, France.

Mathematical Reflections 2 (2016)

S369. Given the polynomial P (x) = xn + a1 xn1 + + an1 x + an having n real roots (not necessarily
distinct) in the interval [0, 1], prove that 3a21 + 2a1 8a22 1.
Proposed by Nguyen Viet Hung, Hanoi University of Science, Vietnam

Solution by Latofat Bobojonova, Academic lyceum S.H.Sirojiddinov, Tashkent, Uzbekistan


Comment. There must be a typo. Counter example: P (x) = x2 1.5x + 0.5.
The original version should be :
Given the polynomial P (x) = xn + a1 xn1 + ... + an1 x + an having n real roots (not necessarily
distinct) in interval [0, 1], prove that 3a21 + 2a1 8a2 1.
Solution. Notice that a1 = (x1 + x2 + ... + xn ) and a2 =

(x1 + x2 + ... + xn )2 (x21 + x22 + ... + x2n )


,
2

so we can rewrite our inequality as



3(x1 + x2 + ... + xn )2 2(x1 + x2 + ... + xn ) 4 (x1 + x2 + ... + xn )2 (x21 + x22 + ... + x2n ) 1,
which is equivalent to
4(x21 + x22 + ... + x2n ) 2(x1 + x2 + ... + xn ) 1 + (x1 + x2 + ... + xn )2 .
Since x1 , x2 , ..., xn [0, 1] we have that x21 + x22 + ... + x2n x1 + x2 + ... + xn , so
4(x21 + x22 + ... + x2n ) 2(x1 + x2 + ... + xn ) 4(x1 + x2 + ... + xn ) 2(x1 + x2 + ... + xn ) =
= 2(x1 + x2 + ... + xn ) (x1 + x2 + ... + xn )2 + 1,
where the last inequality follows from (x1 + x2 + ... + xn 1)2 0. Equality holds when x1 = 1 and
x2 = x3 = ... = xn = 0.
Also solved by Khurshid Juraev, Academic lyceum named after S.H.Sirojiddinov, Tashkent, Uzbekistan;
Nermin Hodzic, Dobosnica, Bosnia and Herzegovina; Yong Xi Wang, Affiliated High School of Shanxi
University.

Mathematical Reflections 2 (2016)

S370. Prove that in any triangle,


2





2
3a 2b2 ma + 3b2 2c2 mb + 3c2 2a2 mc 8K .
R

Proposed by Titu Andreescu, University of Texas at Dallas, USA

Solution by Latofat Bobojonova, academic lyceum S.H.Sirojiddinov, Tashkent, Uzbekistan


Firstly, we observe a useful lemma.
b2 + c2
.
4R
Proof. Let ABC be triangle inscribed to the circle w. Let ma intersect w at X (X 6= A). By power of a
point we have
Lemma. In any triangle it is true that ma

AX = ma +

4m2a + a2
2b2 + 2c2
b2 + c2
a2
=
=
2R ma
.
4ma
4ma
4ma
4R

So, the lemma is proved. Back to the main problem.


Using the lemma we get
|3a2 2b2 |ma + |3b2 2c2 |mb + |3c2 2a2 |mc
c2 + a2
a2 + b2
b2 + c2
+ |3b2 2c2 |
+ |3c2 2a2 |
.
4R
4R
4R
And finally, using the Triangle Inequality and Herons formula gives us the desired result:
|3a2 2b2 |

|3a2 2b2 |(b2 + c2 ) + |3b2 2c2 |(c2 + a2 ) + |3c2 2a2 |(a2 + b2 )


|(3a2 2b2 )(b2 + c2 ) + (3b2 2c2 )(c2 + a2 ) + (3c2 2a2 )(a2 + b2 )| =
= |2(2a2 b2 + 2b2 c2 + 2c2 a2 a4 b4 c4 )| = 2|16K 2 | = 32K 2 .
Also solved by Arkady Alt, San Jose, CA, USA; Nicuor Zlota, Traian Vuia Technical College, Focani,
Romania.

Mathematical Reflections 2 (2016)

10

S371. Let ABC be a triangle and let M be the midpoint of the arc BAC. Let AL be the bisector of angle A,
and let I be the incenter of triangle ABC. Line M I intersects the circumcircle of triangle ABC at K
and line BC intersects the circumcircle of triangle AKL at P . If P I AK = {X}, and KI BC = {Y },
prove that XY k AL.
Proposed by Latofat Bobojonova, Tashkent, Uzbekistan

Solution by Khurshid Juraev, Academic lyceum named after S.H.Sirojiddinov, Tashkent, Uzbekistan
Firstly, we prove the well known lemma. After that we will show KXY = KAI.
Lemma. Let S be the midpoint of minor arc BC. Then KS, BC, and the line through I perpendicular to
AS (let it be l) are concurrent.
Proof. IKS = 2 implies that l is tangent to (IKS), but we know this line is tangent to (BIC), so l is the
radical axis of (IKS) and (BIC). BC is the radical axis of (ABC) and (BIC), and KS is the radical axis
of (ABC) and (IKS), so these lines are concurrent at the radical center of (ABC), (IKS), and (BIC). 
Let the point of concurrency be P 0 . Then
LP 0 K = CBK BKP = CAK BAS = CAK SAC = KAS.
This shows us that BC intersects (AKL) at P 0 , so P = P 0 and the Lemma implies that P I AI. Thus
P XK =

KAL = KP L = P Y K
2
2

So P XY K is cyclc quadrilateral. Finally, KXY = KP Y = KAI. We are done!


Also solved by Nermin Hodzic, Dobosnica, Bosnia and Herzegovina; Nikolaos Evgenidis, M.N. Raptou
High School, Larissa, Greece; Prithwijit De, HBCSE, Mumbai, India; Li Zhou, Polk State College, Winter
Haven, FL, USA.

Mathematical Reflections 2 (2016)

11

S372. Prove that in any triangle,

2
1
(ma mb + mb mc + mc ma ) (a2 + b2 + c2 ) + 3K.
3
4
Proposed by Titu Andreescu, University of Texas at Dallas, USA;

Solution by Nguyen Viet Hung, HSGS, Hanoi University of Science, Vietnam


We have known that there exists a triangle whose side-lengths are ma , mb , mc and its area is K 0 = 43 KABC .
Applying the HadwigerFinsler inequality for this triangle we obtain

m2a + m2b + m2c 4 3K 0 + (ma mb )2 + (mb mc )2 + (mc ma )2


or

3
2(ma mb + mb mc + mc ma ) 4 3 KABC + m2a + m2b + m2c
4
On the other hand, easy to see that
3
m2a + m2b + m2c = (a2 + b2 + c2 )
4
Thus we get

3
2(ma mb + mb mc + mc ma ) 3 3KABC + (a2 + b2 + c2 ),
4
which is equivalent to the desired result, and we are done!
Also solved by David Stoner, Harvard University, Cambridge, MA, USA; Bobojonova Latofat, Academic
lyceum S.H.Sirojiddinov, Tashkent, Uzbekistan; Yong Xi Wang, Affiliated High School of Shanxi University; Adnan Ali, Student in A.E.C.S-4, Mumbai, India; Vicente Vicario Garca, Sevilla, Spain; Nicuor
Zlota Traian Vuia Technical College, Focani, Romania; Khurshid Juraev, Academic lyceum named after
S.H.Sirojiddinov, Tashkent, Uzbekistan; AN-anduud Problem Solving Group, Ulaanbaatar, Mongolia; Evgenidis Nikolaos, M.N. Raptou High School, Larissa, Greece; Nermin Hodzic, Dobosnica, Bosnia and Herzegovina;
Arkady Alt, San Jose, CA, USA.

Mathematical Reflections 2 (2016)

12

Undergraduate problems

U367. Let {an }n1 be the sequence of real numbers given by a1 = 4 and 3an+1 = (an + 1)3 5, n 1. Prove
that an is a positive integer for all n, and evaluate

an 1
.
an + an + 1
2

n=1

Proposed by Albert Stadler, Herrliberg, Switzerland

Solution by Daniel Lasaosa, Pamplona, Spain


Define an = 3bn + 1, or b1 = 1, and for all n 1,
bn+1 = 3bn (bn + 1)2 + bn
therefore since b1 is a positive integer, by trivial induction bn is an integer for all n, hence so is an . Moreover,
an 1
bn (bn + 1)
bn
bn+1 bn
=
=
=
=
2
an + an + 1
bn+1 + 1
3 (bn + 1) (bn+1 + 1)
3bn + 3bn + 1
2

=
or

N
X

1
1

,
3 (bn + 1) 3 (bn+1 + 1)

an 1
1
1
1
1
=

=
,
an + an + 1
3 (b1 + 1) 3 (bN +1 + 1)
6 3 (bN +1 + 1)
2

n=1

and since bN clearly diverges to + because for any positive bn we have bn+1 > 3bn 3 and b1 = 1, we finally
conclude that

X
1
1
1
an 1
= lim
= .
2
an + an + 1
6 N 3 (bN +1 + 1)
6
n=1

Also solved by Yong Xi Wang, Affiliated High School of Shanxi University, Shanxi, China; Brian Bradie,
Christopher Newport University, Newport News, VA, USA; AN-anduud Problem Solving Group, Ulaanbaatar,
Mongolia; Li Zhou, Polk State College, Winter Haven, FL, USA; Adnan Ali, Student in A.E.C.S-4, Mumbai,
India; Arkady Alt, San Jose, CA, USA; Jorge Ledesma, UNAM, Mexico City, Mexico; Miguel Cidr/as Senra,
Pontevedra, Spain; Moubinool Omarjee , Lyc/ee Henri IV , Paris, France; Nermin Hodzic, Dobosnica,
Bosnia and Herzegovina; Kosmidis Dimitrios, National and Kapodistrian University of Athens, Greece.

Mathematical Reflections 2 (2016)

13

U368. Let
xn =
Evaluate limn

r
2+

3
+ +
2

r
n+1

n+1
,
n

n = 1, 2, 3, . . . .

xn
n .

Proposed by Nguyen Viet Hung, Hanoi University of Science, Vietnam

Solution by Li Zhou, Polk State College, USA


For 1 k n, by the binomial theorem,


1
1+
k(k + 1)

k+1
=1+

1
k+1
+ >
.
k
k

Hence,
r
1<
and thus n < xn < n + 1

1
n+1 .

k+1

k+1
1
1
1
<1+
=1+
,
k
k(k + 1)
k k+1

Therefore, by the squeeze theorem, limn

xn
n

= 1.

Also solved by Daniel Lasaosa, Pamplona, Spain; Yong Xi Wang, Affiliated High School of Shanxi University, Shanxi, China; Brian Bradie, Department of Mathematics, Christopher Newport University, Newport
News, VA, USA; Hyun Min Victoria Woo, Northfield Mount Hermon School, Mount Hermon, MA, USA; Ji
Eun Kim, Tabor Academy, Marion, MA, USA; AN-anduud Problem Solving Group, Ulaanbaatar, Mongolia;
Vicente Vicario Garca, Sevilla, Spain; Kosmidis Dimitrios, National and Kapodistrian University of Athens,
Greece; ngel Plaza, Universidad de Las Palmas de Gran Canaria, Spain; Alessandro Ventullo, Milan, Italy;
Arkady Alt, San Jose, CA, USA; Corneliu Mnescu-Avram, Transportation High School, Ploieti, Romania;
David Stoner, Harvard University, Cambridge, MA, USA; Henry Ricardo, New York Math Circle, Tappan,
NY, USA; Joel Schlosberg, Bayside, NY, USA; Miguel Cidr/as Senra, Pontevedra, Spain; Moubinool Omarjee Lyce Henri IV, Paris, France; Nermin Hodzic, Dobosnica, Bosnia and Herzegovina; Albert Stadler,
Herrliberg, Switzerland.

Mathematical Reflections 2 (2016)

14

U369. Prove that

648 X
6217
1
= 2
.
3
3
3
3
35
k (k + 1) (k + 2) (k + 3)
630
k=1

Proposed by Albert Stadler, Herrliberg, Switzerland

Solution by Li Zhou, Polk State College, USA


By partial fraction decomposition,




11 1
1
103 1
1
1

+
[k(k + 1)(k + 2)(k + 3)]3
1296 k k + 3
432 k 2 (k + 3)2




1
9
1
1
1
1

216 k 3 (k + 3)3
16 k + 1 k + 2




3
1
1
1
1
1
+
+

.
16 (k + 1)2 (k + 2)2
8 (k + 1)3 (k + 2)3
Hence,

X
k=1

1
k 3 (k + 1)3 (k + 2)3 (k + 3)3





1 1
11 2
103
1
1
1+ +

1 2 2
1296
2 3
432 3
2
3


 
1
1
9 1
1
1+ 3 + 3
+
216
2
3
16 2
 2

 


3
1
1 1
35
6217
2
+
11 2
=

.
16 3
2
8 23
648
630

Also solved by Daniel Lasaosa, Pamplona, Spain; Yong Xi Wang, Affiliated High School of Shanxi University; Brian Bradie, Department of Mathematics, Christopher Newport University, Newport News, VA, USA;
Adnan Ali, Student in A.E.C.S-4, Mumbai, India; Corneliu Mnescu-Avram, Transportation High School,
Ploieti, Romania; Joel Schlosberg, Bayside, NY, USA; Jorge Ledesma, Faculty of Sciences UNAM, Mexico
City, Mexico; Moubinool Omarjee Lyce Henri IV, Paris, France; Nermin Hodzic, Dobosnica, Bosnia and
Herzegovina.

Mathematical Reflections 2 (2016)

15

U370. Evaluate

lim

1 + 2x

x0

1 + 3x
x

1 + nx 1

Proposed by Nguyen Viet Hung, Hanoi University of Science, Vietnam

Solution by Alessandro Ventullo, Milan, Italy


Observe that for any k = 2, . . . , n and for any x such that |x| < 1, we have

k
Hence,

1 + 2x

1
1 + kx = 1 + kx + o(x2 ) = 1 + x + o(x2 ).
k

1 + 3x . . .

Therefore,

lim

x0

1 + 2x

1 + nx 1 = (1 + x + o(x2 ))n1 1
= (1 + (n 1)x + o(x2 )) 1
= (n 1)x + o(x2 ).

1 + 3x . . .
x

1 + nx 1

= n 1.

Also solved by Daniel Lasaosa, Pamplona, Spain; Yong Xi Wang, Affiliated High School of Shanxi University; Brian Bradie, Department of Mathematics, Christopher Newport University, Newport News, VA,
USA; Hyun Min Victoria Woo, Northfield Mount Hermon School, Mount Hermon, MA, USA; Ji Eun Kim,
Tabor Academy, Marion, MA, USA; AN-anduud Problem Solving Group, Ulaanbaatar, Mongolia; Vicente
Vicario Garca, Sevilla, Spain; Li Zhou, Polk State College, Winter Haven, FL, USA; Kosmidis Dimitrios,
National and Kapodistrian University of Athens, Greece; Adnan Ali, Student in A.E.C.S-4, Mumbai, India;
ngel Plaza, Department of Mathematics, University of Las Palmas de Gran Canaria, Spain; Arkady Alt,
San Jose, CA, USA; Corneliu Mnescu-Avram, Transportation High School, Ploieti, Romania; David Stoner, Harvard University, Cambridge, MA, USA; Henry Ricardo, New York Math Circle, Tappan, NY, USA;
Joel Schlosberg, Bayside, NY, USA; Moubinool Omarjee Lyce Henri IV, Paris, France; Nermin Hodzic,
Dobosnica, Bosnia and Herzegovina; Albert Stadler, Herrliberg, Switzerland.

Mathematical Reflections 2 (2016)

16

U371. Let n be a positive integer, and let An = (aij ) be the n n matrix where aij = x(i+j2) , x being a
variable. Evaluate the determinant of An .
Proposed by Mehtaab Sawhney, Commack High School, New York, USA

Solution by Daniel Lasaosa, Pamplona, Spain


Denote by D(n) the n n determinant, and simplify its calculation through the following transformation
(which clearly leaves the value of the determinant unchanged):
2

Step 1 For i = n, n 1, . . . , 2 and in this order, subtract x2i3 = x(i1) (i2) times row i 1 from row i.
2
2
2
Therefore, in position (i, j) for i 2, we have x(i+j2) x(i+j3) +2i3 = x(i+j3) +2i3 x2(j1) 1 .
For j = 1, this means that all elements of the first column are zero except for (1, 1), which is 1.
2

Step 2 Subtract x(j1) times the resulting first column from column j for j = 2, 3, . . . , n, or all elements in
the first row and the first column are zero except for (1, 1), which is 1. Moreover, all elements such
that i 2 remain unchanged in this step, or their values are as after Step 1.




Step 3 Extract from column j a factor x2(j1) 1 , or a prefactor of x2 1 x4 1 x2(n1) 1 is
2
added in front of the determinant, while position (i, j) for i, j 2 is now occupied by x(i+j3) +2i3 =
2
x(i+j4) +2(2i+j5) .
Step 4 Extract from column j a factor x2(j1) and from row i a factor x4(i2) for i, j 2, or position (i, j) for
i, j 2 is now occupied by the same term as position (i 1, j 1) in the (n 1) (n 1) determinant,
and a prefactor of x4(0+1++n2)+2(1+2++n1) = x(n1)(3n4) is added in front of the determinant.
All the first row and all the first column are still occupied by zeros, except for position (1, 1) which is
occupied by 1.
From the previous transformation, it is clear that
D(n) = x(n1)(3n4)

n1
Y


x2i 1 D(n 1)

i=1
2

Trivially, D(1) = x(1+12) = 1, or after a simple inductive process, we conclude that


D(n) = x

n(n1)2

n1
Y

x2i 1

ni

i=1

Also solved by AN-anduud Problem Solving Group, Ulaanbaatar, Mongolia; Li Zhou, Polk State College,
Winter Haven, FL, USA; Kosmidis Dimitrios, National and Kapodistrian University of Athens, Greece;
Problem Solving Group, Department of Financial Engineering, University of the Aegean, Mitilini, Greece;
Nermin Hodzic, Dobosnica, Bosnia and Herzegovina.

Mathematical Reflections 2 (2016)

17

U372. Let , > 0 be real numbers, and let f : R R be a continuous function such that f (x) 6= 0, for all
x in a neighborhood U of 0. Evaluate
R x
t f (t)dt
lim R0x
.
f (t)dt
x0+
t
0
Proposed by Nguyen Viet Hung, Hanoi University of Science, Vietnam

Solution by Alessandro
Milan, ItalyR
R x Ventullo,
x

Observe that limx0+ 0 t f (t) dt = limx0+ 0 t f (t) dt = 0. Let g(t) = t f (t) and h(t) = t f (t). We
have
Z x
d
g(t) dt = g(x) = (x) f (x)
dx 0
and
d
dx
Clearly,

h(t) dt = h(x) = (x) f (x).

d R x
h(t) dt 6= 0 for all x in a neighborhood U of 0 (x 6= 0). We have
dx 0

d R x

if >
0
0 t f (t) dt
dx
lim
=
1
if =

x0+ d R x

t
f
(t)
dt
+ if < .
dx 0

By LHpitals Rule we conclude that

0
dt
0
= 1
lim R

x0+ x t f (t) dt

0
+
R x

t f (t)

if >
if =
if < .

Also solved by Daniel Lasaosa, Pamplona, Spain; Vicente Vicario Garca, Sevilla, Spain; Li Zhou, Polk
State College, Winter Haven, FL, USA; Kosmidis Dimitrios, National and Kapodistrian University of Athens,
Greece; ngel Plaza, Department of Mathematics, University of Las Palmas de Gran Canaria, Spain; Henry
Ricardo, New York Math Circle, Tappan, NY, USA; Moubinool Omarjee Lyce Henri IV, Paris, France;
Nermin Hodzic, Dobosnica, Bosnia and Herzegovina.

Mathematical Reflections 2 (2016)

18

Olympiad problems

O367. Prove that for any positive integer a > 81 there are positive integers x, y, z such that
a=

x3 + y 3
.
z 3 + a3
Proposed by Nairi Sedrakyan, Yerevan, Armenia

Solution by Li Zhou, Polk State College, USA


We show that the claim is true for all integers a 82, 4 a 8, and 25 a 71. First, if x = 3n(a 3n3 ),
y = 9n4 , and z = 9n3 a, then


(3n)3 (a 3n3 )3 + (3n3 )3
x3 + y 3
=
= a.
z 3 + a3
(9n3 a)3 + a3
Thus, it remains onlyp
to show that
for each a above, there exists positive integer n such that a 3n3 > 0 and
p
a
a
3
9n a > 0, that is, 3 9 < n < 3 3 . Indeed, for 4 a 8, n = 1 works; and for 25 a 71, n = 2 works.
p
Finally, if a 82, then there exists k 3 such that 3k 3 < a 3(k + 1)3 . Take n = k. Clearly, n < 3 a3 .
q
p
3
3
Moreover, 3k 3 (k + 1)3 = 2k 2 (k 3) + 3k(k 1) 1 > 0, that is, 3 a9 (k+1)
< k = n, completing the
3
proof.
Also solved by Moubinool Omarjee Lyce Henri IV, Paris, France; Albert Stadler, Herrliberg, Switzerland.

Mathematical Reflections 2 (2016)

19

O368. Let a, b, c, d, e, f be real numbers such that a + b + c + d + e + f = 15 and a2 + b2 + c2 + d2 + e2 + f 2 = 45.


Prove that abcdef 160.
Proposed by Marius Stanean, Zalau, Romania

First solution by Evgenidis Nikolaos, M.N. Raptou High School, Larissa, Greece
We are going to use the Lagrange Multipliers method to solve the problem. Let g1 = a + b + c + d + e + f 15
and g2 = a2 + b2 + c2 + d2 + e2 + f 2 45. Then Lagranges function is
F = abcdef (a + b + c + d + e + f 15) (a2 + b2 + c2 + d2 + e2 + f 2 45).
For the first partial derivatives of the function we have
Fa0 = bcdef 2a,
Fb0 = acdef 2b,
Fc0 = abdef 2c,
Fd0 = abcef 2d,
Fe0 = abcdf 2e,
Ff0 = abcde 2f.
Since we want every partial derivative to be equal to zero, the above relations give
(b a)(cdef + 2) = 0
(c a)(bdef + 2) = 0
(d a)(bcef + 2) = 0
(e a)(bcdf + 2) = 0
(f a)(bcde + 2) = 0.
Now we have to examine separately some cases.
Case 1: all variables are equal (a = b = c = d = e = f ).
This case is easily rejected since the given hypothesis cannot be satisfied simultaneously.
Case 2: 5 variables are equal, say a = b = c = d = e. Then 2 = bcde.
(
5a + f = 15
The given hypothesis give
. This system of equations has two solution a = 3, f = 0
5a2 + f 2 = 45
and a = 2, f = 5 for which values we find F = 0 and F = 160, respectively.
Case 3: 4 variables are equal, say a = b = c = d. Then bcdf = bcde e = f .
(
4a + 2f = 15
Now we have to solve the following system of equations
4a2 + 2f 2 = 45

5
following pair of solutions a = 15+2
, b = 154
6
6
values, we can find that F < 160 in both cases.

Mathematical Reflections 2 (2016)

and a =

152 5
,b
6

. The system has the

15+4 5
.
6

Plugging in these

20

Case 4: 3 variables are equal, say a = b = c. Then bcef = bcdf = bcde d = e = f .


As before, we have
( to solve the following system of equations that are deduced from the initial conditions

3a + 3f = 15
5 5
5 5
5+ 5
5+ 5
of the problem
.
The
solutions
are
a
=
,
b
=
and
a
=
,
b
=
2
2
2
2 .
3a2 + 3f 2 = 45
For these values it is F = 125.
Hence, we have proved that the maximum value of F is F = 160 or equivalently abcdef 160, as we wished
to prove.

Mathematical Reflections 2 (2016)

21

Second solution by the author


From Cauchy-Schwarzs inequality, we have
5(45 f 2 ) = 5(a2 + b2 + c2 + d2 + e2 ) (a + b + c + d + e)2 = (15 f )2
225 5f 2 225 30f + f 2 f (f 5) 0 0 f 5,
and similarly we obtain that a, b, c, d, e [0, 5].
The inequality is symmetric so we can assume, without losing generality of problem, that 0 a b
c d e f 5 and let m = a + b, n = c + d, p = e + f = 0 m n p and m + n + p = 15.
We denote x = ab, y = cd and z = ef = 0 x y z.
2
2
2
Now we fix m, n, p (as constants) and let x, y, z vary (as variables) such that x m4 , y n4 , z p4
2
2
2
and x + y + z = m +n 2+p 45 . In this case we must find maximum of xyz. The problem can be divided into
four cases:
2

1. m4 n4
cant happen.

p2
4

m2 +n2 +p2 45
6
2

x+y+z
3

which means x = y = z = a = b = c = d = e = f what

2. m4 m +n 6+p 45 p4 such that m4 + 2n4 m +n 2+p 45 m2 + 2p2 90.


I will prove the following
Lemma 1. If 0 x y z such that x + y + z = s and x a 3s , y b and a + 2b s then the
maximum of xyz is obtained for x = a, y = b and z = s a b.
Proof. Indeed we have from AM-GM inequality
2 2

a b (s a b) xyz = b(s a b)xa(s a b)yabz

b(s a b)x + a(s a b)y + abz


3

3
=

[b(s 2a b)x + a(s a 2b)y + ab(x + y + z)]3


[b(s 2a + b)a + a(s a 2b)b + abs]3

=
27
27
a3 b3 (s a b)3 = xyz ab(s a b).

2 2

+2p 90)
According to lemma we have xyz m n (m +n
.
64
2
2
2
2
2
Denote f (m, n, p) = m n (m + n + 2p 90) and I will prove that


m+n m+n
f (m, n, p) f
,
, p
2
2


(m + n)6
(m + n)4
2 2
2
2
2
2 2
m n (m + n ) + (2p 90)
m n 0
32
16


(m n)2 m4 + 8m3 n 2m2 n2 + 8mn3 + n4
2
2
2
+ (2p 90)(m + 6mn + n ) 0
16
2


(m n)2 (m2 + n2 )(m2 + 6mn + n2 ) + 2mn(m n)2
2
2
2
+ (2p 90)(m + 6mn + n ) 0
16
2

which is true because




(m n)2 (m2 + n2 )(m2 + 6mn + n2 )
2
2
2
LHS
+ (2p 90)(m + 6mn + n )
16
2


(m n)2 (m2 + 6mn + n2 ) m2 + n2
2
+ 2p 90
16
2
(m n)2 (m2 + 6mn + n2 )(m2 + 2p2 90)
0.
16
Mathematical Reflections 2 (2016)

22

Further we must show that f (m, m, p) 45 10 where 2m + p = 15 and m2 + 2p2 90 0. So


2 90 0 3m2 40m + 120 0 m
+ p h= 15= mi 5 and m2 + 2(15
2m)
h3m 2m
h
i
i
0, 2023 10 20+23 10 , . Therefore m 0, 2023 10 and then we have to show that
m4 (2m2 + 2(15 2m)2 90) 45 10
 2

m (m 6) 32 (m 4)2 (m + 2) 0
which is obviously true, with the equality when m = n = 4(x = y = 4) = a = b = c = d = 2 and
p = 7 = e = 2, f = 5.
2

3. m4 m +n 6+p 45 p4 such that m4 + 2n4 m +n 2+p 45 m2 + 2p2 90.


I will prove the following
Lemma 2. If 0 x y z such that x + y + z = s and x a 3s , y b and a + 2b s then the
maximum of xyz is obtained for x = a and y = z = sa
2 .
Proof. In this case we have
xyz x

(y + z)2
(s x)2
=x
.
4
4

We define the function f : [0, a] 7 R, f (x) = x(s x)2 . We have f 0 (x) = (s x)(s 3x) 0 for x [0, a].
Therefore the maximum of f is obtained for x = a and y = z = sa
2 b.
2

According to lemma we have that the maximum of xyz is obtained for x = m4 and y = z =
But from y = z it follows that c = d = e = f so n = p and then remains to show that

m2 +2n2 +2p2 90
.
8

m2 (m2 + 4p2 90)2 46 10


2
2
where m + 2p = 15 and m2 + 2p2 90. So
2m +
3m
2m + (15 m) 180
 p = 15 = m  5 and
2
0 m 10m + 15 0 m 5 10, 5 + 10 . Therefore m 5 10, 5 and then we have to
show that
m2 (m2 + (15 m)2 90)2 46 10

m(2m2 30m + 135) 64 10.

Let
the
function
g
:
[5

10,
5] 7 R, g(m) = m(2m2 30m + 135). We have
g 0 (m) = 6m2 60m + 135 =




6 m 5 + 210 m 5 210 , so the maximum value of g is for m0 = 5 210 , but g(m0 ) = 175 + 5 10 <

64 10 175 < 59 10 which is true.

4.

m2 +n2 +p2 45
6

m2
4 ,

but this case can not be held because this is equivalent with
m2 + 90 2n2 + 2p2

a2 + 2ab + b2 + 90 2c2 + 4cd + 2d2 + 2e2 + 4ef + 2f 2


a2 + b2 + 2ab + 90 2(45 a2 b2 ) + 4cd + 4ef
3(a2 + b2 ) + 2ab 4cd + 4ef
and this is true only if a = b = c = d = e = f , what cant happen, because 3a2 3cd, 3b2 3ef and
2ab cd + ef .
Also solved by Moubinool Omarjee Lyce Henri IV, Paris, France.

Mathematical Reflections 2 (2016)

23

O369. Let a, b, c > 0. Prove that


a2 + bc b2 + ca c2 + ab p 2
+
+
3(a + b2 + c2 ).
b+c
c+a
a+b
Proposed by An Zhen-Ping, Xianyang Normal University, China

Solution by Arkady Alt, San Jose, CA, USA


Let p := ab + bc + ca, q := abc and let a + b + c = 1(due to homogeneity).
Then
X a2 + bc


1 X 2
1 X 2
=
a + bc (1 b) (1 c) =
a + bc (a + bc) =
b+c
p q cyc
p q cyc
cyc

1
a3 + b3 + c3 + 3abc + abc (a + b + c) + a2 b2 + b2 c2 + c2 a2 =
pq
1 + 3q 3p + 3q + q + p2 2q
1 + 5q 3p + p2
=
pq
pq
and inequality becomes
1 3p + p2 + 5q p
3(1 2p).
pq
1
(a + b + c)2
(1 p) (4p 1)
(because ab + bc + ca
) and q
3
3
6
P 2
a (a b) (a c) 0 in 1-p-q notation).
(Schure Inequality

Also note that 0 < p

cyc



1 3p + p2 + 5q
(1 p) (4p 1)
Since
increasing in q > 0 and q max 0,
pq
6
then
(1 p) (4p 1)
1 3p + p2 + 5
1 3p + p2 + 5q
1 + 7p 14p2
6

=
(1 p) (4p 1)
pq
4p2 + p + 1
p
6
and suffices to prove inequalities:
1 + 7p 14p2 p
3(1 2p)
4p2 + p + 1

1 + 7p 14p2

2

3(1 2p) 4p2 + p + 1

2

for p [1/4, 1/3]


and
1 3p + p2 p
3(1 2p) for p (0, 1/4].
p

Mathematical Reflections 2 (2016)

24

For p [1/4, 1/3] we have 1 + 7p 14p2


2 (3p 1) 2p2 + 5p 1

2

3(1 2p) 4p2 + p + 1

2




8p2 p 1 = 2 (1 3p) 2p2 + 5p 1 1 + p 8p2 0

because

 2
1
3
1
+5 1=
2p + 5p 1 2
4
4
8

2
and for p (0, 1/4] we have 1 + 7p 14p2 3p2 (1 2p) = 196p4 190p3 + 18p2 + 14p + 1 =
196p4 190p3 + 18p2 + 14p + 1 = 1 4p + 18p (1 4p) + 90p2 (1 4p) + 170p3 + 196p4 > 0.
2

Also solved by Yong Xi Wang, Affiliated High School of Shanxi University, Shanxi, China; AN-anduud
Problem Solving Group, Ulaanbaatar, Mongolia; WSA; Adnan Ali, Student in A.E.C.S-4, Mumbai, India;
Ahn, Young Seob, Seoul, Korea; David Stoner, Harvard University, Cambridge, MA, USA; Duy Quan Tran,
University of Health Science, Ho Chi Minh City, Vietnam; Khurshid Juraev, Academic lyceum named after
S.H.Sirojiddinov, Tashkent, Uzbekistan; Bobojonova Latofat, Academic lyceum S.H.Sirojiddinov, Tashkent,
Uzbekistan; Moubinool Omarjee Lyce Henri IV, Paris, France; Nermin Hodzic, Dobosnica, Bosnia and
Herzegovina; Nguyen Viet Hung, Hanoi University of Science, Vietnam; Evgenidis Nikolaos, M.N. Raptou
High School, Larissa, Greece; Paul Revenant, Lyce Champollion, Grenoble, France.

Mathematical Reflections 2 (2016)

25

O370. For any positive integer n we denote by S(n) the sum of digits of n. Prove that for any integer n such
that gcd(3, n) = 1 and any integer k, k > S(n)2 + 7S(n) 9, there exists an integer m such that n|m
and S(m) = k.
Proposed by Nairi Sedrakyan, Yerevan, Armenia

Solution by the author



.
Without loss of generality we can suppose that (n, 10) = 1. Let M = S(m)|m..n , where S(m)denotes the
sum of the m numbers digits. Lets now prove the following properties.
.
.
Property 1. If a;b M, then a + b M. Indeed, let a = S(A), b = S(B) and A, B ..n, then 10k A + B ..m
and S(10k A + B) = S(A) + S(B) = a + b M, where k is the number of digits in B.
Corollary 1. If a,b M and x,y are non negatived integers, then ax + by M. it is evident that
S(n) M.
Property 2. If (n, 10) = 1, then S(n) + 9 M. Indeed, for two numbers in 1, 10, 102 , ..., 10n with the
same remainder after division by n, their difference 10i 10j = 10j (10ij 1) will contain n, from which we
.
.
find that 10ij 1..n(i > j), so there exists a natural number k such that 10k 1..n.
Now let n = al al1 ...a1 . Consider the following number
m=

10i1 10a1 +...+ai1+1)k + ... + 10a1 +a2 +...+ai1+ai )k )

ai A

+10l1 (10a1 +...+ai1+1)k + ... + 10a1 +a2 +...+al1+al 1)k )


+10l2 (10a1 +a2 +...+al+1)k + ... + 10a1 +a2 +...+al1+al +10)k ),
where A are nonzero digits from a1 a2 ...aal1 . It is easy to see that S(m) = a1 +a2 +...+al1 +al 1+10 =
S(n) + 9 and
m a1 + 10a2 + ... + 10l2 al1 + 10l1 (al 1) + 10l2 10

(mod n)

.
hence m..n. So S(n) + 9 M.
Lemma. If a, b N and (a, b) = 1, then any number c > ab a b can be represented as ax + by, where
x and y are non negative integers.
Indeed, since (a, b) = 1, then there exist integer numbers u and v such that au + bv = 1. Without loss
of generality we can suppose that 0 < u b. Indeed, suppose that pb < u (p + 1)b, then in that case
a(u pb) + b(v + pa) = 1.
Now, let c = ab a b + d, where d > 0, then from the above proven it follows that d may be represented
in the form ax1 + by1 , where 0 < x1 b, consequently c = a(x1 1) + b(a + y1 1). If a + y1 1 < 0, then
d = ax1 + by1 ax1 + b(a) ab ab = 0, which is impossible.
The lemma is proved.
Since (n, 3) = 1, then (S(n), 3) = 1 and (S(n), S(n) + 9) = 1, consequently by the corollary, properties
and lemma for any
k > S(n)(S(n) + 9 S(n) (S(n) + 9) = S 2 (n) + 7S(n) 9.

Mathematical Reflections 2 (2016)

26

O371. Let ABC(AB 6= BC) be a triangle, and let D and E be the feet of altitudes from B and C, respectively.
Denote by M, N, P , the midpoints of BC, M D, M E, respectively. If {S} = N P BC and T is the
point of intersection of DE with the line through A, which is parallel to BC, prove that ST is tangent
to the circumcircle of triangle ADE.
Proposed by Marius Stnean, Zalu, Romnia

Solution by Adnan Ali, Student in A.E.C.S-4, Mumbai, India


It is well-known that M D and M E are tangents to (ADE). It is also clearly seen that the line through A,
parallel to BC is also tangent to (ADE) at A. Let (ADE) AM = K 6= A. Then because T lies on the
polar of M w.r.t (ADE) and T A is tangent to (ADE) at A, by La Hires Theorem, we conclude that
T K is tangent to (ADE) at K.
Thus, it is sufficient to prove that SK is tangent to (ADE) at K. Denote by H, the orthocenter of
4ABC and let DE BC = F . Then we claim that H is the orthocenter of 4AM F . The proof of our claim
is as follows:
Proof : Consider the cyclic quadrilateral BEDC. Then from Brocards Theorem, M , the center of (BEDC)
is the orthocenter of 4HAF . So, M H AF and we also know by definition that AH M F . Thus, H is
the orthocenter of 4AM F , completing the proof of our claim.
Since AH is the diameter of (AEHD), HKA = 90 and so F, H, K are collinear. Also from the
converse of midpoint theorem, S is the midpoint of M F . Moreover F KA = 90 4F KM is rightangled at K. So S is the circumcenter of 4F KM and we have SKF = SF K = M F H. But we
also have M AH = 90 LM A = 90 F M K = M F K = M F H, where L = AH BC. Thus,
SKF = SF K = M F H = M AH = KAH. Denote by G, the midpoint of AH. Then KAH =
GAK = GKA GKA = SKF . This implies that GKS = AKF , and because AKF = 90 , we
conclude that SK is tangent to (ADE) at K, completing our proof.
Also solved by David Stoner, Harvard University, Cambridge, MA, USA; Khurshid Juraev, Academic lyceum named after S.H.Sirojiddinov, Tashkent, Uzbekistan; Bobojonova Latofat, Academic lyceum
S.H.Sirojiddinov, Tashkent, Uzbekistan; Evgenidis Nikolaos, M.N. Raptou High School, Larissa, Greece.

Mathematical Reflections 2 (2016)

27

O372. A regular n-gon b of side b is drawn inside a regular n-gon a of side a such that the center of the
circumcircle of a is not inside b . Prove that
b<

a
2 cos2

.
2n

Proposed by Nairi Sedrakyan, Yerevan, Armenia

Solution by Evgenidis Nikolaos, M.N. Raptou High School, Larissa, Greece

It is well known that 2 cos2 2n


= cos n + 1.
Also by the Law of Sines in any triangle Ai Aj Ak and Bi Bj Bk it is obvious that a = 2Ra sin n and b =
2Rb sin n , where Ra , Rb are the circumradius of a , b , respectively and Ai , Aj , Ak , Bi , Bj , Bk are vertices
of the polygons with 1 i 6= j 6= k n.
Then, the given inequality can be written equivalently in the following way
a
b<
2 cos2

2n

2Ra sin n

2Rb sin <


Rb (cos + 1) < Ra .
n
cos n + 1
n

However, it is obvious that Rb cos n is the apothem, hb , of regular n-gon, b . Hence, it suffices to prove that
Ra > Rb + hb . Then, we will prove that the circumcircle of b is inside or tangent to the circumcircle of a ,
since that would be equivalent to
Ra Rb + Oa Ob (1),
and it is easy to see that Oa Ob > hb , as Oa does not lie inside b , where Oa , Ob are the circumcenters of
a , b respectively..
Assume for the sake of contradiction, that the two circumcircles intersect. Let K be a point (not a vertex)
lying on a minor arc Am Al of a and inside the circumcircle of b , where m, l are consecutive vertices of the
regular n-gon.
Then, it is Am KAl = (n1)
. Furthermore, a is convex and b lies inside it, so Am KAl > Bi KBj
n
for any vertices Bi , Bj of b . But Bi KBj (n1)
, since K lies inside the circumcircle of b , which is a
n
(n1)
contradiction since we should have Am KAl > n , obviously false.
Therefore, the two circumcircles do not intersect and the result follows from (1).
Also solved by Bobojonova Latofat, Academic lyceum S.H.Sirojiddinov, Tashkent, Uzbekistan; Khurshid
Juraev, Academic lyceum named after S.H.Sirojiddinov, Tashkent, Uzbekistan.

Mathematical Reflections 2 (2016)

28

Você também pode gostar